Lesson: Strengthen the Argument Questions

Comment on Strengthen the Argument Questions

Thank you for designing this amazing preparation tool.
I notice that in weaken/strengthen the argument questions, we analyse which "premise" is weakening or strengthening the argument. is it possible that we have to look for an assumption rather than a premise ?
Best,
Sage
gmat-admin's picture

Yes, you can look for assumptions too. Remember that an assumption is just an unstated premise that must be true. So, either way, you're looking for a type of premise.
More here: https://www.gmatprepnow.com/module/gmat-critical-reasoning/video/1133

I notice a question stem that seems to be fitting several of the question types. The question stem is "Which of the following most logically completes the argument" - and it's so 'Strengthen The Argument' questions as well as "Assumption' and 'Structure' questions -- what is the best way to differentiate? Thanks!
-Yvonne
gmat-admin's picture

It depends on the argument passage ends.

In some cases, the argument is missing a conclusion. So, the question is a Conclusion/Inference question in disguise.

Other times, it can be a Flawed Argument question. For example, "Despite Joe's thoughtful points, his conclusion his incorrect because __"

And so on. In fact, questions that ask "Which of the following most logically completes the argument?" can be pretty much any question type. It all depends on the passage that needs completing.

Hi Brent- is this strengthen the argument question? Doesn't sound so from question stem

https://gmatclub.com/forum/which-of-the-following-most-logically-completes-the-argument-49838.html
gmat-admin's picture

Question linK: https://gmatclub.com/forum/which-of-the-following-most-logically-complet...

Yes, it's a strengthen the argument question.

The conclusion is that "planting spring wheat will be more profitable than planting winter wheat."

The question stem asks us to identify information that supports this conclusion.

Hi Brent,

In below question I did shortlist option E but I felt it talks about targeted advertisement where as conclusion refers to targeted individuals. Targeted advertisement can be set to larger set of individuals which are not targeted
individuals.

Please explain.
https://gmatclub.com/forum/advertising-by-mail-has-become-much-less-effective-with-fewer-consume-201330.html

gmat-admin's picture

Question link: https://gmatclub.com/forum/advertising-by-mail-has-become-much-less-effe...

I think the test-makers are treating "target individuals" and "targeted advertising" as equivalent. This was my impression as well.

By the way, Mitch (GMATGuruNY) provides a nice analysis of the question here: http://www.beatthegmat.com/2016-og-cr-60-t291151.html

Hi Brent.

Is this a strengthen question? How can I recognize it? Thanks.

https://gmatclub.com/forum/the-united-states-government-uses-only-a-household-s-cash-102079.html
gmat-admin's picture

QUESTION STEM: Which of the following, if true, validates the contention that the government's calculation methods must be altered in order to provide statistics that measure true poverty?

In this case, the question stem doesn't clearly state the question type. However, the opening words, "Which of the following, if true...", are typical of either a Strengthen or Weaken the Conclusion question. That said, we won't really know for sure until we read the actual passage:

"The United States government uses only a household's cash income before taxes to determine whether that household falls below the poverty line in a given year; capital gains, non-cash government benefits, and tax credits are not included. HOWEVER, YEARLY CASH INCOME IS NOT A FOOL-PROOF MEASURE OF A GIVEN HOUSEHOLD'S DISPOSABLE INCOME. For example, retirees who live off of capital gains from an extensive portfolio could earn hundreds of thousands of dollars, yet be classified by the government as living in "poverty" because this income is not included in the calculation."

Okay, so the passage has the conclusion: Yearly cash income is NOT a fool-proof measure of a given household's disposable income. This conclusion seems to IMPLY that "the government's calculation methods must be altered in order to provide statistics that measure true poverty" (the quoted words appear in the question stem).

So, yes, we are trying to strengthen the implied conclusion that "the government's calculation methods must be altered in order to provide statistics that measure true poverty"

Does that help?

Cheers,
Brent

https://gmatclub.com/forum/studies-in-restaurants-show-that-the-tips-left-by-customers-who-pay-th-200170.html
sir in this question i wasnt able to find answer but got answer by eliminating other options although answer is correct but i cant understand how option b strengthens the argument
gmat-admin's picture

Question link: https://gmatclub.com/forum/studies-in-restaurants-show-that-the-tips-lef...

The original argument suggests that there's a relationship between getting reminded of one's potential spending power (by seeing a credit card logo) and tip size.

That is, when you have A LOT of spending power (i.e., potential credit) then you leave BIG tips.

Answer choice B suggests that, when you have VERY LITTLE spending power then you leave SMALL tips.

So, answer choice B strengthens the general relationship between spending power (via credit card) and tip size.

Does that help?

Cheers,
Brent

https://gmatclub.com/forum/the-spacing-of-the-four-holes-on-a-fragment-of-a-bone-flute-excavated-59671.html
please explain
gmat-admin's picture

Question link: https://gmatclub.com/forum/the-spacing-of-the-four-holes-on-a-fragment-o...

Have you reviewed GMATNinja's response (https://gmatclub.com/forum/the-spacing-of-the-four-holes-on-a-fragment-o...)? I can't think of a better explanation.

Once you take a look, let me know if you have any questions.

Cheers,
Brent

https://gmatclub.com/forum/antarctic-seals-dive-to-great-depths-and-stay-submerged-for-hours-235675.html
sir how can 'a' be strength it just say horses store we dont know how it is related to seals
gmat-admin's picture

Question link: https://gmatclub.com/forum/antarctic-seals-dive-to-great-depths-and-stay...

The hypothesis is that, for long dives seals store oxygenated blood in their spleens.

Answer choice A tells us that it's possible for another living organism (a horse) to can store oxygenated blood in its spleens for use during exertion.

So, if one living organism (a horse) can do it, then this supports that hypothesis that a seal can also do it.

Does that help?

Cheers,
Brent

https://gmatclub.com/forum/political-theorist-newly-enacted-laws-need-a-period-of-immunity-286290.html
sir though i was able to get this one correct im not able reject option e on solid reason please help.
gmat-admin's picture

Hi Brent - another one from me. I really don't get the charred bone from campfire question here: https://gmatclub.com/forum/in-swartkans-territory-archaeologists-discovered-charred-bone-fragmen-138894.html

I don't see why E is the correct answer over C
gmat-admin's picture

Question link: https://gmatclub.com/forum/in-swartkans-territory-archaeologists-discove...

Here's my take:

The analysis tells us that the bones COULD have been heated by a campfire.
However, it's also possible that the bones were heated by a forest fire.

That's all the information we have.

We have two possible explanations for the charred bones:
1) campfire made by early hominids
2) naturally occurring forest fire

At the moment, there's nothing to suggest that one explanation is better than the other. So, let's examine the answer choices.

C) The bone fragments were fitted together by the archaeologists to form the complete skeletons of several animals.
This doesn't really strengthen the argument, since bones charred in a forest fire could still be fitted together to form complete skeletons

(E) The bone fragments were found in several distinct layers of limestone that contained primitive cutting tools known to have been used by early hominids.

This helps strengthen the connection between the charred bones to early hominids.
As such, E is the best answer.

Does that help?

Cheers,
Brent

In parts of South America, vitamin-A deficiency is a serious health problem, especially among children. In one region, agriculturists are attempting to improve nutrition by encouraging farmers to plant a new variety of sweet potato called SPK004 that is rich in beta-carotene, which the body converts into vitamin A. The plan has good chances of success, since sweet potato is a staple of the region’s diet and agriculture, and the varieties currently grown contain little beta-carotene.

Which of the following, if true, most strongly supports the prediction that the plan will succeed?

(A) The growing conditions required by the varieties of sweet potato currently cultivated in the region are conditions in which SPK004 can flourish.

(C) There are no other varieties of sweet potato that are significantly richer in beta-carotene than SPK004 is.
------------------------------------------------------------------------
A Keep - Good Assumption: That both have same cultivating conditions
C Already Stated as:
Premise: SPK is rich in Beta-C
OptC: Significantly richer in Beta-C

C: CORRECT - REASON: Is Re-stating a premise with superlative degree is a better answer than an Assumtion???
gmat-admin's picture

Your reasoning is perfect. The correct IS A.
See: https://gmatclub.com/forum/in-parts-of-south-america-vitamin-a-deficienc...

Cheers,
Brent

Oh! Thanks, sorry for the trouble.
gmat-admin's picture

No trouble at all :-)

Many industrialized nations are trying to reduce atmospheric concentrations of carbon dioxide, a gas released by the burning of fossil fuels. One proposal is to replace conventional cement, which is made with calcium carbonate, by a new “eco-cement.” This new cement, made with magnesium carbonate, absorbs large amount of carbon dioxide when exposed to the atmosphere. Therefore, using eco-cement for new concrete building projects will significantly help reduce atmospheric concentrations of carbon dioxide.

Which of the following, if true, most STRENGTHENS the argument? 

B [Supporting or elaborating on an existing premise] - (B) Eco-cement is strengthened when absorbed carbon dioxide reacts with the cement. 
D [Adding a new supporting premise] -> The manufacture of eco-cement uses considerably less fossil fuel per unit of cement than the manufacture of conventional cement does.

I chose B assuming that Elaborating an existing Premise is better choice than to add a new information that talks about MANUFACTURING.

An option adding new info is more relevant of an answer than Supporting an existing Premise? Is there some priority pattern too?
gmat-admin's picture

The #1 priority is to identify and summarize the CONCLUSION.

CONCLUSION: Using eco-cement will significantly help reduce concentrations of CO2.

Answer choice B provides another (and totally separate) positive feature about eco-cement (it reduces CO2 AND it is super strong)
This does not strengthen the conclusion that eco-cement will significantly help reduce concentrations of CO2.

Answer choice D says that eco-cement's manufacturing process uses less CO2 than the manufacturing process for conventional cement.
So, eco-cement ABSORBS a bunch of CO2, AND its manufacturing process uses less CO2.
This premise definitely strengthens the conclusion that eco-cement will significantly help reduce concentrations of CO2

Cheers,
Brent

Hi!
Brent,
I'm able to narrow down on 2 options in Critical reasoning and only 62.5% times(as per GMAT CLUB ERROR LOG) I'm able to select correct answer. Also I'm taking at about 3:30 to 4:30 Mins to answer a question. Most of my time goes into understanding the Arguement. Any tips?
gmat-admin's picture

If you're still early in your prep, don't worry about your timing (content first, speed second).

I suggest that you focus on ONE question type at a time.
For example, if you answer 50 Assumption questions in a row, you'll learn a great deal about the various ways the GMAT can test you on this one question type.

Keep at it. Your accuracy WILL improve.

Cheers,
Brent

Petrochemical industry officials have said that the extreme pressure exerted on plant managers during the last five years to improve profits by cutting costs has done nothing to impair the industry’s ability to operate safely. However, environmentalists contend that the recent rash of serious oil spills and accidents at petrochemical plants is traceable to cost-cutting measures.

Which of the following, if true, would provide the strongest support for the position held by industry officials?


Conclusion: Petrochemical industry officials have said that the extreme pressure exerted on plant managers during the last five years to improve profits by cutting costs has done nothing to impair the industry’s ability to operate safely. --> Support this

(C) Despite major cutbacks in most other areas of operation, the petrochemical industry has devoted more of its resources to environmental and safety measures in the last five years than in the preceding five years. ---> And yet safety didnt workout and Managers of Petro industry also complained just as Environmentalists. ---> SOUNDS GOOD BUT Against Premise, bcuz It was also Petro Indus Officials who said Managers were under pressure for past 5 years and cost cutting di nothing but impair industry's ability to operate safely.
gmat-admin's picture

Tough question!!

This is a CAUSE AND EFFECT argument.

The petrochemical officials say: Our cost-cutting measures have NOT made us less safe.

The environmentalists say: Your cost-cutting measures HAVE made you less safe. Look at the spills and accidents.

Regardless of how much money is spent on safety, accidents still happen.
So, for example, let's say the petrochemical industry QUADRUPLED the money it spends on safety, and shortly thereafter there was a spill. Can we say that the spending increase CAUSED the spill? No, there may be other reasons.

When students first read the passage, many will assume that the cost-cutting measures probably include cutting costs on safety measures.

C) Despite major cutbacks in most other areas of operation, the petrochemical industry has devoted MORE of its resources to environmental and safety measures in the last five years than in the preceding five years.

The increased spending in environmental and safety measures suggests that the industry is likely more safe these days. This strengthens the petrochemical officials' position.

Cheers,
Brent

https://gmatclub.com/forum/the-spacing-of-the-four-holes-on-a-fragment-of-a-bone-flute-excavated-59671.html

need help with this
gmat-admin's picture

Question link: https://gmatclub.com/forum/the-spacing-of-the-four-holes-on-a-fragment-o...

Consider this analogous argument:
Researchers found a PORTION of a shirt from 1587, and this shirt fragment had 4 buttons.
CONCLUSION: People have been wearing SEVEN-BUTTON shirts at least as early as 1587.

What would strengthen the conclusion?

If we could show that the ORIGINAL shirt (from which the portion came) had enough room for 7 buttons, this would strengthen the conclusion.
This is what answer choice E (the correct answer) is saying.

Does that help?

Cheers,
Brent

Hi Brent,

Could you please explain the question below?
https://gmatclub.com/forum/the-irradiation-of-food-kills-bacteria-and-thus-retards-spoilage-howe-138471.html

Thanks in advance!

Hi Brent,

Could you please explain why the choice B is wrong and C is correct?

Is this that B(there are some definite flaws) is incorrect and C is correct

or just because C is better answer than B

https://gmatclub.com/forum/telomerase-is-an-enzyme-that-is-produced-only-in-cells-that-are-active-31920.html

Thank you in advance,
gmat-admin's picture

Link: https://gmatclub.com/forum/telomerase-is-an-enzyme-that-is-produced-only...

This is an Inference question (for more on this question type, watch: https://www.gmatprepnow.com/module/gmat-critical-reasoning/video/1140)

(B) In children, the ONLY body tissues from which telomerase is ABSENT are those in which cells are not rapidly dividing.
The passage tells us that Telomerase is produced only in cells that are actively dividing, but doesn't say that Telomerase is produced in ALL cells that are actively dividing. As such Telomerase may not be present in cells that are actively dividing.

Cheers,
Brent

Hi Brent,

Is it strengthen the argument question and if yes, why B is the correct answer,

I don't get why by sending " livestock to market much earlier than they otherwise would." would cause a short-term decrease in price.

Thank you in advance
gmat-admin's picture

Link: https://gmatclub.com/forum/escalating-worldwide-demand-for-corn-has-led-...

Note: Please post the link to the question.

This is a Paradox question.
The paradox: If livestock farmers leave the business, this will result in a shortage in the supply of meat, so meat prices should INCREASE. However, the passage states that "observers expect an immediate short-term DECREASE in meat prices."

What could explain this apparent paradox?

(B) Generally, farmers who are squeezed out of the livestock business send their livestock to market much earlier than they otherwise would.
Ahhh! If these livestock farmers (faced with going out of business) bring their their livestock to market earlier than usual, there will be a short-terms abundance of meat on the market, which will result in a short-term DECREASE in meat prices.

Does that help?

Cheers,
Brent

Hi Brent,

Would you agree with me that this answer is the best among given answers, but not the ideal(A)

For me, if red-winged blackbirds are first to stop in spring it doesn't mean that they will eat all the rice and thus absorb all poison prepared by farmers. There is no evidence that they did not leave any rice as a leftover so that other types of birds could eat it and therefore be in danger.

Sorry for so many questions,

Thank you for your patience,
gmat-admin's picture

Link: https://gmatclub.com/forum/each-year-red-winged-blackbirds-stop-in-a-cer...

Note: Please post the link to the question.

You are answering a different question. The conclusion doesn't state that this plan will be effective, and the question stem doesn't mention anything about whether the blackbirds will eat the rice.
----------------------------
The residents are concerned that the poisoned rice could threaten certain species of rare migratory birds.
Despite this, the wildlife agency approved the permits.

The question: Which of the following, if true, most helps to justify the wildlife agency’s approval of the permits, given the concerns voiced by some residents?

We need an answer that somehow addresses the residents' concerns.

(A) In the region where the red-winged blackbirds stop, they are the first birds to be present in the spring.
If the blackbirds are the first birds to show up in the spring, then you can place set out the poisoned rice in the early spring (BEFORE the rare migratory birds arrive.) Then, you can remove the poisoned rice before the rare migratory birds arrive.

Does that help?

Cheers,
Brent

Hi Brent,

What is the difference between the argument and the conclusion?

Thank you in advance
gmat-admin's picture

In theory, an argument is comprised of premises, assumptions and a conclusion.

However, in practice, "argument" and "conclusion" are often used interchangeably.

The strength of an argument is based on how well the conclusion logically follows from the premises.

So, to "strengthen the conclusion" is to make the conclusion more logically follow from the premises.
Likewise, to strengthen the argument" is to make the conclusion more logically follow from the premises.

I hope that helps.

Cheers,
Brent

Hi Brent,

In your breakdown of question from offical Guide 2016 this question was classified as "strengthen the argument question". Are you sure that this is not a resolve paradox type of question?

https://gmatclub.com/forum/between-1980-and-2000-the-sea-otter-population-of-the-aleutian-islands-202574.html

Thank you in advance,
gmat-admin's picture

Question link: https://gmatclub.com/forum/between-1980-and-2000-the-sea-otter-populatio...

This is definitely a strengthen the argument question.
The question stem reads "Which of the following, if true, most strengthens the argument?"

In the given passage, there is no indication of a paradox that needs to be resolved.

Cheers,
Brent

Hi Brent,

Thank you,

Now I understand,

I somehow conclude that A is correct answer:

Snowmaking machines work by spraying a mist that freezes immediately on contact with cold air. Because the sudden freezing kills bacteria, QuickFreeze is planning to market a wastewater purification system that works on the same principle. The process works only when temperatures are cold, however, so municipalities using it will still need to maintain a conventional system.

Which of the following, if true, provides the strongest grounds for a prediction that municipalities will buy QuickFreeze's purification system despite the need to maintain a conventional purification system as well?

A) Bacteria are not the only impurities that must be removed from wastewater.
-> As the QF kills only bacteria whereas we need conventional systems to kill other impurities found in water. Thus, this statement clears that we need conventional system for other impurities and QF for bacteria (as the only thing that it does).

C) Conventional wastewater purification systems have not been fully successful in killing bacteria at cold temperatures.

This tells us that CONV systems are unable to kill bacteria but doesn't tell whether we need to keep CONV systems or not
gmat-admin's picture

Be careful. There's nothing in the passage that suggests QuickFreeze kills ONLY bacteria. For all we know, QuickFreeze might kill everything in the water.

Likewise, there's nothing in the passage that suggests conventional systems are able to deal with other impurities found in water. For all we know, neither system is good for dealing with impurities other than bacteria.

For these reasons, answer choice A does not help us.

Answer choice C, on the other hand, nicely explains why a municipality would maintain two types of systems. They can use QuickFreeze when temperatures are cold, and they can use conventional systems when the temperatures are not cold.

Does that help?

Hi Brent. Good Afternoon.
In 'Reinforcement Activities' question#5 - 'Snowmaking machines work by spraying a mist that freezes immediately ...', I am confused between option 'C' & 'D'. Can you please explain how 'D' doesn't fit in as a solution?
gmat-admin's picture

Question link: https://gmatclub.com/forum/snowmaking-machines-work-by-spraying-a-mist-t...

The question asks us to find a reason why municipalities would buy QuickFreeze's purification system WHILE ALSO maintaining a conventional purification system. In other words, why would a city bother having/maintaining two different purification systems?

(C) Conventional wastewater purification systems have not been fully successful in killing bacteria at cold temperatures.
If conventional purification systems can't kill bacteria at cold temperatures, it makes perfect sense for a municipality to use QuickFreeze during the winter, and use a conventional purification system when the temperatures are warmer. That way, the city gets safe drinking water all year round.

(D) During times of warm weather, when it is not in use, QuickFreeze's purification system requires relatively little maintenance.
This just tells us that, once in awhile, QuickFreeze's purification system doesn't require much maintenance. That's not a compelling reason to have two purification systems. Why not just have ONE purification system?

Thank you so much. It clarifies.

Pages

Office Hours

On December 20, 2023, Brent will stop offering office hours. 

Change Playback Speed

You have the option of watching the videos at various speeds (25% faster, 50% faster, etc). To change the playback speed, click the settings icon on the right side of the video status bar.

Have a question about this video?

Post your question in the Comment section below, and a GMAT expert will answer it as fast as humanly possible.

Free “Question of the Day” emails!